Question on Mean Value Theorem & Intermediate Value Theorem

Click For Summary
The discussion revolves around proving the equation ∫₀⁴ f(t) dt = 2[αf(α) + βf(β)] for 0 < α, β < 2. The initial approach involves defining functions g(x) and h(x) and applying the Intermediate Value Theorem, but the validity of the proof is questioned. It is pointed out that the assumption made does not hold universally for all integrable functions f, as the relationship does not account for values of f on the interval (2, 4]. The conversation concludes with the realization that the original question posed by the teacher may have been incorrect. The discussion highlights the complexities involved in applying the Mean Value and Intermediate Value Theorems in this context.
Titan97
Gold Member
Messages
450
Reaction score
18

Homework Statement


for ##0<\alpha,\beta<2##, prove that ##\int_0^4f(t)dt=2[\alpha f(\alpha)+\beta f(\beta)]##

Homework Equations


Mean value theorem: ##f'(c)=\frac{f(b)-f(a)}{b-a}##

The Attempt at a Solution


I got the answer for the question but I have made an assumption but I don't know if it's correct.

Let ##g(x)=\int_0^{x^2}f(t)dt## and let ##h(x)=g'(x)=2xf(x^2)##

now, applying intermediate value theorem for ##h(x)## in ##x\in (\alpha,\beta)##, there exists a point ##x=k## such that,
$$h(k)=\frac{h(\alpha)+h(\beta)}{2}=\alpha f(\alpha)+\beta f(\beta)$$
$$g'(k)=\alpha f(\alpha)+\beta f(\beta)$$
assumption: Now, for g(x), if ##x=k## also satisfies Mean value theorem and since ##0<k<2##,

$$g'(k)=\frac{f(2)-f(0)}{2}=\frac{1}{2}\int_0^{4}f(t)dt=\alpha f(\alpha)+\beta f(\beta)$$

Hence, $$\int_0^{4}f(t)dt=2[\alpha f(\alpha)+\beta f(\beta)]$$
 
Last edited by a moderator:
Physics news on Phys.org
What, exactly, are you to prove? You say "for 0&lt;α,β&lt; 2 \int_0^4 f(t)dt=2[\alpha f(\alpha)+\beta f(\beta)]"
That reads "for all \alpha and \beta strictly between 0 and 2, and for any integrable function, f" this equality is true.

But that is clearly NOT true. In your "proof" you appear to determine that there exist numbers \alpha and \beta that work.
 
Titan97 said:

Homework Statement


for ##0<\alpha,\beta<2##, prove that ##\int_0^4f(t)dt=2[\alpha f(\alpha)+\beta f(\beta)]##

What exactly is the question? To prove that there exist 0 &lt; \alpha &lt; 2 and 0 &lt; \beta &lt; 2 such that <br /> \int_0^4 f(t)\,dt = 2(\alpha f(\alpha) + \beta f(\beta))? Are there any conditions on f?

Homework Equations


Mean value theorem: $$f'(c)=\frac{f(b)-f(a)}{b-a}

The Attempt at a Solution


I got the answer for the question but I have made an assumption but I don't know if it's correct.

Let ##g(x)=\int_0^{x^2}f(t)dt## and let ##h(x)=g'(x)=2xf(x^2)##

now, applying intermediate value theorem for ##h(x)## in ##x\in (\alpha,\beta)##, there exists a point ##x=k## such that,
$$h(k)=\frac{h(\alpha)+h(\beta)}{2}=\alpha f(\alpha)+\beta f(\beta)$$

Unfortunately
\frac{h(\alpha) + h(\beta)}{2} = \alpha f(\alpha^2) + \beta f(\beta^2) which doesn't help you.

EDIT: It occurs to me that \alpha f(\alpha) + \beta f(\beta) doesn't involve the value of f on (2, 4] at all, which seems suspicious.
 
Last edited:
I am very sorry. My teacher gave me a wrong question.
 
Question: A clock's minute hand has length 4 and its hour hand has length 3. What is the distance between the tips at the moment when it is increasing most rapidly?(Putnam Exam Question) Answer: Making assumption that both the hands moves at constant angular velocities, the answer is ## \sqrt{7} .## But don't you think this assumption is somewhat doubtful and wrong?

Similar threads

Replies
4
Views
2K
Replies
1
Views
1K
Replies
8
Views
2K
  • · Replies 1 ·
Replies
1
Views
1K
  • · Replies 1 ·
Replies
1
Views
1K
  • · Replies 11 ·
Replies
11
Views
2K
Replies
2
Views
2K
Replies
3
Views
2K
  • · Replies 1 ·
Replies
1
Views
1K
  • · Replies 12 ·
Replies
12
Views
1K